0 Daumen
160 Aufrufe

Aufgabe:

1)

Bestimmen Sie die Menge der \( z \in \mathbb{C} \), für welche die folgenden Potenzreihen konvergieren. In (d) reicht es, den Konvergenzradius der Potenzreihe zu berechnen.
(a) \( \sum \limits_{n \geq 0} \frac{3^{n} \sqrt{n}}{\sqrt{n^{7}+1}} z^{n} \),
(b) \( \sum \limits_{n \geq 0}\left(\frac{1}{2}+\frac{1}{2 n+1}\right)^{n} z^{2 n} \),
(c) \( \sum \limits_{n \geq 0} 2^{-n} 3^{(-1)^{n} n} z^{3 n} \),
(d) \( \sum \limits_{n \geq 0}\left(1+\frac{1}{n+1}\right)^{n^{2}(n+1)} z^{\left(n^{2}\right)} \).


2)

(a) Sei \( \left(a_{n}\right) \) ein beschränkte Folge mit \( \liminf _{n \rightarrow \infty}\left|a_{n}\right|>0 \). Bestimmen Sie den Konvergenzradius der Reihe \( \sum \limits_{n} a_{n} z^{n} . \)

(b) Es sei \( \sum \limits_{k \geq 0} a_{k} \) eine divergente Reihe in \( (0, \infty) \) und die Potenzreihe \( \sum \limits_{k \geq 0} a_{k} z^{k} \) besitze den Konvergenzradius 1. Wir definieren
\( f_{n}:=\sum \limits_{k=0}^{\infty} a_{k}\left(1-\frac{1}{n}\right)^{k} \quad(n \in \mathbb{N}) . \)
Zeigen Sie, dass \( \left(f_{n}\right) \) uneigentlich gegen \( \infty \) konvergiert für \( n \rightarrow \infty \).
Hinweis: Verwenden Sie die Bernouillische Ungleichung, um Terme der Form \( \left(1-\frac{1}{n}\right)^{k} \) nach unten abzuschätzen.

(c) Zeigen Sie, dass die folgenden Grenzwerte existieren und bestimmen Sie deren Wert.
(i) \( \lim \limits_{x \rightarrow 0} \frac{x^{2}}{|x|} \)
(ii) \( \lim \limits_{x \rightarrow \infty}\left(\sqrt{x^{2}+\alpha x+\beta}-x\right) \)
In (ii) seien hierbei \( \alpha, \beta \in \mathbb{R} \) mit \( \alpha, \beta \geq 0 \).


Problem/Ansatz:


Corona bedingt sind die Vorlesungen online. Und ich verstehe garnichts. Wie löse ich diese beiden Übungsaufgaben ?

Danke im Voraus.

Gruß

Löwenzahn

Avatar von

1 Antwort

0 Daumen

Zu 1) Lies in deinem Skript oder in anderen Quellen was zum

Thema Konvergenzradius . z.B.

https://de.wikipedia.org/wiki/Konvergenzradius#Bestimmung_des_Konvergenzradius

Und probiere die Methoden aus. z.B. Quotientenkriterium im Fall (a).

Da ist \( a_n = \frac{3^n \cdot \sqrt{n}}{\sqrt{n^7+1}}  \) und   \( a_{n+1} = \frac{3^{n+1} \cdot \sqrt{n+1}}{\sqrt{(n+1)^7+1}}  \)

Der Quotient ist also (Betrag entfällt, da alles positiv.):

\(  \frac{3^n \cdot \sqrt{n}\cdot \sqrt{(n+1)^7+1}}{\sqrt{n^7+1}\cdot 3^{n+1} \cdot \sqrt{n+1}}  \)

Davon der Grenzwert für n gegen unendlich ist 1/3. Das ist also der

Konvergenzradius. Also konvergiert die Reihe für alle z miz |z| < 1/3.

Für |z|=1/3 musst du extra schauen: Da kürzen sich 3^n und |z|^n und

der Rest ist eine Reihe, bei der die Beträge der Summanden nicht gegen

0 gehen, also konvergiert sie nicht.

Avatar von 288 k 🚀

Ein anderes Problem?

Stell deine Frage

Willkommen bei der Mathelounge! Stell deine Frage einfach und kostenlos

x
Made by a lovely community